Thank you 8, You are really helpful.


所有跟贴·加跟贴·新语丝读书论坛

送交者: cornbug 于 2007-09-10, 16:59:58:

回答: Since you are so serious, I'll start the solution for you 由 008 于 2007-09-10, 15:24:30:

Not an easy math solution for a simple model.

Right now I just don't know how to solve d(d(theta)/dt)/dt = 0 for the maximal angular velocity at x and t. Forgot about all math but I will solve it. Graphically looks like at the crest and trough the tangential angle of theta with respect to horizon has a fastest rate change approaching zero.






所有跟贴:


加跟贴

笔名: 密码: 注册笔名请按这里

标题:

内容: (BBCode使用说明